\lecture{17}{2023-12-14}{Silver's Theorem} We now want to prove \yaref{thm:silver}. \gist{% \begin{remark} The hypothesis of \yaref{thm:silver} is consistent with $\ZFC$. \end{remark} }{} We will only prove \gist{% \yaref{thm:silver} in the special case that $\kappa = \aleph_{\omega_1}$ (see \yaref{thm:silver1}). The general proof differs only in notation. }{\yaref{thm:silver1}.} \gist{% \begin{remark} It is important that the cofinality is uncountable. For example it is consistent with $\ZFC$ that $2^{\aleph_n} = \aleph_{n+1}$ for all $n < \omega$ but at the same time $2^{\aleph_{\omega}} = \aleph_{\omega + 2}$. \end{remark} }{} \begin{yarefproof}{thm:silver1} \gist{% We need to count the number of $X \subseteq \aleph_{\omega_1}.$ Let us fix $\langle f_\lambda : \lambda < \kappa \text{ an infinite cardinal} \rangle$ such that $f_{\lambda}\colon \cP(\lambda) \to \lambda^+$ is bijective for each $\lambda < \kappa$. For $X \subseteq \aleph_{\omega_1}$ define \begin{IEEEeqnarray*}{rCl} f_X\colon \omega_1 &\longrightarrow & \aleph_{\omega_1} \\ \alpha &\longmapsto & f_{\aleph_\alpha}(X \cap \aleph_\alpha). \end{IEEEeqnarray*} \begin{claim} For $X,Y \subseteq \aleph_{\omega_1}$ it is $X \neq Y \iff f_X \neq f_Y$. \end{claim} \begin{subproof} $X \neq Y$ holds iff $X \cap \aleph_\alpha \neq Y \cap \aleph_\alpha$ for some $\alpha < \omega_1$. But then $f_X(\alpha) \neq f_Y(\alpha)$. \end{subproof} For $X, Y \subseteq \aleph_{\omega_1}$ write $X \le Y$ iff \[ \{\alpha < \omega_1 : f_X(\alpha) \le f_Y(\alpha)\} \] is stationary. \begin{claim} For all $X,Y \subseteq \aleph_{\omega_1}$, $X \le Y$ or $Y \le X$. \end{claim} \begin{subproof} Suppose that $X \nleq Y$ and $Y \nleq X$. Then there are clubs $C,D \subseteq \omega_1$ such that \[ C \cap \{\alpha < \omega_1 : f_X(\alpha) \le f_Y(\alpha)\} = \emptyset \] and \[ D \cap \{\alpha < \omega_1 : f_Y(\alpha) \le f_X(\alpha)\} = \emptyset. \] Note that $C \cap D$ is a club. Take some $\alpha \in C \cap D$. But then $f_X(\alpha) \le f_Y(\alpha)$ or $f_{Y}(\alpha) \le f_X(\alpha)$ $\lightning$ \end{subproof} \begin{claim} \label{thm:silver:p:c3}. Let $X \subseteq \aleph_{\omega_1}$. Then \[ |\{Y \subseteq \aleph_{\omega_1} : Y \le X\}| \le \aleph_{\omega_1}. \] \end{claim} \begin{subproof} Write $A \coloneqq \{Y \subseteq X_{\omega_1} : Y \le X\}$. Suppose $|A| \ge \aleph_{\omega_1 + 1}$. For each $Y \in A$ we have that \[ S_Y \coloneqq \{\alpha : f_Y(\alpha) \le f_X(\alpha)\} \] is a stationary subset of $\omega_1$. Since by assumption $2^{\aleph_1} = \aleph_2$, there are at most $\aleph_2$ such $S_Y$. Suppose that for each $S \subseteq \omega_1$, \[ |\{Y \in A : S_Y = S\}| < \aleph_{\omega_1 + 1}. \] Then $A$ is the union of $\le \aleph_2$ many sets of size $< \aleph_{\omega_1 + 1}$. Thus this is a contradiction since $\aleph_{\omega_1 + 1}$ is regular. So there exists a stationary $S \subseteq \omega_1$ such that \[ A_1 = \{Y \subseteq \aleph_{\omega_1} : S_Y = S\} \] has cardinality $\aleph_{\omega_1 + 1}$. We have \[f_Y(\alpha) \le f_X(\alpha) = f_{\aleph_\alpha}(X \cap \aleph_\alpha)< \aleph_{\alpha + 1}\] for all $Y \in A_1, \alpha \in S$. Let $\langle g_{\alpha} : \alpha \in S \rangle$ be such that $g_\alpha\colon \aleph_{\alpha} \twoheadrightarrow f_X(\alpha) + 1$ is a surjection for all $\alpha \in S$. Then for each $Y \in A_1$ define \begin{IEEEeqnarray*}{rCl} \overline{f}_Y\colon S &\longrightarrow & \aleph_{\omega_1} \\ \alpha &\longmapsto & \min \{\xi : g_\alpha(\xi) = f_Y(\alpha)\}. \end{IEEEeqnarray*} Let $D$ be the set of all limit ordinals $< \omega_1$. Then $S \cap D$ is a stationary set: If $C$ is a club, then $C \cap D$ is a club, hence $(S \cap D) \cap C = S \cap (D \cap C) \neq \emptyset$. Now to each $Y \in A$ we may associate a regressive function \begin{IEEEeqnarray*}{rCl} h_Y \colon S \cap D &\longrightarrow & \omega_1 \\ \alpha &\longmapsto & \min \{\beta < \alpha : \overline{f}_Y(\alpha) < \aleph_{\beta}\}. \end{IEEEeqnarray*} $h_Y$ is regressive, so by \yaref{thm:fodor} there is a stationary $T_Y \subseteq S \cap D$ on which $h_Y$ is constant. By an argument as before, there is a stationary $T \subseteq S \cap D$ such that \[ |A_2| = \aleph_{\omega_1 +1}, \] where $A_2 \coloneqq \{Y \in A_1 : T_Y = T\}$. Let $\beta < \omega_1$ be such that for all $Y \in A_2$ and for all $\alpha \in T$, $h_Y(\alpha) = \beta$. % TODO WHY DOES THIS WORK? Then $\overline{f}_Y(\alpha) < \aleph_\beta$ for all $Y \in A_2$ and $\alpha \in T$. There are at most $\aleph_\beta^{\aleph_1}$ many functions $T \to \aleph_\beta$, but \begin{IEEEeqnarray*}{rCl} \aleph_\beta^{\aleph_1} &\le & 2^{\aleph_\beta \cdot \aleph_1}\\ &=& \aleph_{\beta+1} \cdot \aleph_2\\ &<& \aleph_{\omega_1}. \end{IEEEeqnarray*} Suppose that for each function $\tilde{f}\colon T \to \aleph_\beta$ there are $< \aleph_{\omega_1 + 1}$ many $Y \in A_2$ with $\overline{f}_Y \cap T = \tilde{f}$. Then $A_2$ is the union of $<\aleph_{\omega_1}$ many sets each of size $< \aleph_{\omega_1 + 1}$ $\lightning$. Hence for some $\tilde{f}\colon T \to \aleph_\beta$, \[ |A_3| = \aleph_{\omega_1 + 1}, \] where $A_3 = \{Y \in A_2 : \overline{f}_Y\defon{T} = \tilde{f}\}$. Let $Y, Y' \in A_3$ and $\alpha \in T$. Then \[ \overline{f}_Y(\alpha) = \overline{f}_{Y'}(\alpha), \] hence \[ f_{\aleph_\alpha}(Y \cap \aleph_\alpha) = f_Y(\alpha) = f_{Y'}(\alpha) = f_{\aleph_\alpha}(Y' \cap \aleph_\alpha), \] i.e.~$Y \cap \aleph_\alpha = Y' \cap \aleph_\alpha$. Since $T$ is cofinal in $\omega_1$, it follows that $Y = Y'$. So $|A_3| \le 1 \lightning$ \end{subproof} Let us now define a sequence $\langle X_i : i < \aleph_{\omega_1 + 1} \rangle$ of subsets of $\aleph_{\omega_1 + 1}$ as follows: Suppose $\langle X_j : j < i \rangle$ were already chosen. Consider \[ \{Y \subseteq \aleph_{\omega_1} : \exists j < i.~Y \le X_j\} = \bigcup_{j < i} \{Y \subseteq \aleph_{\omega_1} : Y \le X_j\}. \] This set has cardinality $\le \aleph_{\omega_1}$ by \yaref{thm:silver:p:c3}. Let $X_i \subseteq \aleph_{\omega_1}$ be such that $X_i \nleq X_j$ for all $j < i$. The set \[ P \coloneqq \{Y \subseteq \aleph_{\omega_1} : \exists i < \aleph_{\omega_1 + 1} .~Y \le X_i\} = \bigcup_{i < \aleph_{\omega_1 + 1}} \{Y \subseteq \aleph_{\omega_1} : Y \le X_i\} \] has size $\le \aleph_{\omega_1 + 1}$ (in fact the size is exactly $\aleph_{\omega_1 + 1}$). On the other hand $P = \cP(\aleph_{\omega_1})$ because if $X \subseteq \aleph_{\omega_1}$ is such that $X \nleq X_i$ for all $i < \aleph_{\omega_1 + 1}$, then $X_i \le X$ for all $i < \aleph_{\omega_1 + 1}$, so such a set $X$ does not exist by \yaref{thm:silver:p:c3}. }{Need to count $X \subseteq \aleph_{ \omega_{1}}$. \begin{itemize} \item Fix bijections $f_\lambda\colon 2^{\lambda} \to \lambda^+$ for all infinite cardinals $\lambda < \kappa$. \item For $X \subseteq \aleph_{ \omega_1}$ define $f_X\colon \omega_1 \to \aleph_{ \omega_1}, \alpha \mapsto f_{\aleph_\alpha}(X \cap \aleph_\alpha)$. \item (1) $X \neq Y \iff f_X \neq f_Y$: \begin{itemize} \item $X \neq Y \iff \exists \alpha.~X \cap \aleph_\alpha \neq Y \cap \aleph_\alpha \iff \exists \alpha.~f_X(\alpha) \neq f_Y(\alpha)$. \end{itemize} \item $X \le Y :\iff \{\alpha < \omega_1 : f_X(\alpha) \le f_Y(\alpha)\}$ stationary. \item (2) $X\le Y \lor Y \le X$: Suppose $X \not\le Y, Y \not\le X$. Choose witnessing clubs $C, D$. $C \cap D$ is club, so $C\cap D \ni \alpha \implies f_X(\alpha) \substack{\nleq\\\ngeq} f_Y(\alpha) \lightning$ \item (3) $X \subseteq \aleph_{ \omega_1}$, then $|\underbrace{\{Y \subseteq \aleph_{ \omega_1} : Y \le X\}}_{A}| \le \aleph_{ \omega_1}$ \begin{itemize} \item Suppose $|A| \ge \aleph_{ \omega_1 + 1}$. \item $S_Y \coloneqq \{\alpha : f_Y(\alpha) \le f_X(\alpha)\}$ stationary for all $Y \in A$. $2^{\aleph_1} = \aleph_2 \implies$ at most $\aleph_2$ such $S_Y$. \item If $\forall S \in \cP( \omega_1).~ |\underbrace{\{Y \in A : S_Y = S\}}_{A_S}| < \aleph_{ \omega_1 + 1}$, then $|A| \le \aleph_2 \cdot <\aleph_{ \omega_1 + 1}$ $\lightning$ $\aleph_{ \omega_1 + 1}$ regular. \item So $\exists S \in \omega_1.~|A_S| = \aleph_{ \omega_1 + 1}$. \item Fix surjection $\langle g_\alpha : \aleph_\alpha \twoheadrightarrow f_X(\alpha) + 1 : \alpha \in S \rangle$. ($f_Y(\alpha) \le f_X(\alpha) < \aleph_{\alpha+1}$) \item $\forall Y \in A_S$ define $\overline{f}_Y \colon S \to \aleph_{ \omega_1}, \alpha \mapsto \min \{\xi : g_\alpha(\xi) = f_Y(\alpha)\}$. \item $S^\ast$ ($S \cap$ limit ordinals) is stationary. \item $\forall Y \in A$ define $h_Y\colon S^\ast \to \omega_1, \alpha \mapsto \min \{\beta < \alpha : \overline{f}_Y(\alpha) < \aleph_\beta\}$. \item Apply \yaref{thm:fodor} to $h_Y, S^\ast$ to get $T_Y \subseteq S^\ast$ stationary with $h_Y\defon{T_Y}$ constant. \item $\exists T.~|\underbrace{\{Y \in A_S : T_Y = T\}}_{A_{S,T}}| = \aleph_{ \omega_1 + 1}$. \item Let $\{\beta\} = h_Y''T$, i.e.~$\overline{f}_Y(\alpha) < \aleph_{\beta}$ for $Y\in A_{S,T}, \alpha \in T$. \item $\leftindex^T \aleph_\beta \le 2^{\aleph_\beta \cdot \aleph_1} = \aleph_{\beta+1} \aleph_2 < \aleph_{ \omega_1}$. \item $\exists \tilde{f}\colon T \to \aleph_\beta .~ |\underbrace{\{Y \in A_{S,T} : \overline{f}_Y\defon{T} = \tilde{f}\} }_{A_{S,T,\tilde{f}}}| = \aleph_{ \omega_1 + 1}$. \item $Y,Y' \in A_{S,T,\tilde{f}} \implies \forall \alpha \in T.~f_Y(\alpha) = f_{Y'}\left( \alpha \right) \overset{T \text{ unbounded}}{\implies} Y = Y'$ Thus $|A_{S,T, \tilde{f}} | \le 1 \lightning$. \end{itemize} \item Define sequence $\langle X_i : i < \aleph_{ \omega_1 + 1} \rangle$ of subsets of $\aleph_{\omega_1}$: \begin{itemize} \item Consider $\{Y \subseteq \aleph_{ \omega_1} : \exists j < i.~Y \le X_j\}$ (cardinality $ \le \aleph_{ \omega_1}$), Take $X_i \subseteq \aleph_{ \omega_1}$ such that $X_i \subseteq \aleph_{ \omega_1}$ such that $X_i \not\le X_j$ for all $j < i$. \item $P \coloneqq \{Y \subseteq \aleph_{ \omega_1} : \exists i < \aleph_{ \omega_1 + 1}.~Y \le X_i\}$ \end{itemize} \item $|P| \overset{\text{in fact } =}{\le} \aleph_{ \omega_1 + 1} \aleph_{ \omega_1} = \aleph_{ \omega_1 + 1}$ by (3). \item $X \in \cP(\aleph_{ \omega_1}) \setminus P \implies \forall i < \aleph_{ \omega_1 + 1}.~X_i \le X$ $\lightning$ (3). Thus $P = \cP(\aleph_{ \omega_1})$. \end{itemize} } \end{yarefproof}